Стационарный растворный узел: Стационарные растворные узлы — купить по низким ценам

Содержание

Растворный узел для удобрений своими руками

  • и.

    Существует такое, что для ; В i ( x ,  P )=0 для .

    ii.

    Для , x В i ( x ,  P )∈ C 900 06 1 ([0,  l i )).

    iii.

    Для любого r >0 существует V r >0 такое, что для ( x ,  P ), , | Р | Н , .

  • и.

    Существует неубывающая функция такая, что для x и .

    ii.

    Существует неубывающая функция такая, что для всех x ∈[0,  l i ) и .

    iii.

    Существует неубывающая функция такая, что для всех x ∈[0,  l i ), P , с | Р | Н , .

  • и.

    Существует неубывающая функция такая, что для всех х , , .

    ii.

    Существует неубывающая функция такая, что для всех x ∈[0,  l i ), .

    iii.

    Существует неубывающая функция такая, что для всех x ∈[0,  l i ), P , причем | Р | Н , .

    iv.

    Существует положительная константа такая, что для всех x ∈[0,  l i ), .

а.

f(x)≠λx для всех x∈C σ таких, что ∥x∥=σ и для всех λ>1.

б.

Существуют τ∈(0, σ) и x 1 ∈C, x 1 ≠0 такие, что x−f(x)≠λx 1 для всех x∈C σ таких, что ∥x∥=τ и для всех λ>0.

i.

λ 2 (0)>1, λ 1 (0)≠1 и (или

9 0257

ii.

существует r>0 таких что λ 2 (P)<1 для всех удовлетворяющих |P| 2 ≥r. Тогда  существует ненулевое стационарное решение уравнения ( 1 ).

Доказательство

Рассмотрим отображение, определяемое уравнением (10), т.е. для и ), где Q ( P ) – матрица, определяемая уравнением и – матрица, определяемая уравнением (11).

Поскольку , , В i (0,  P ) непрерывны в P , f ( P ,  a ) является непрерывным. Заметим, что справедлива следующая оценка: где фигурирует в предположении (G)(iv). Тогда из предположения (F)(ii) имеем Let с , и пусть . Оценка (12) показывает, что f ( C σ ) ограничено и, следовательно, имеет компактное замыкание в .

Чтобы показать, что выполняется условие (a) в предложении 4.1, пусть ( P ,  a )∈ C σ такое, что |( P ,  a 9 0006 )| 4 =σ и предположим, что существует λ>1 такое, что f ( P ,  a )=λ( P ,  a ). Затем обратите внимание, что a i = 0 тогда и только тогда, когда P i = 0 по уравнению (13). Таким образом, вектор A = ( A 1 , A 2 ) должен быть ненулевым, а отношение (14) подразумевает, что A — это собственное вектор, связанное с λ> 1. Тогда по предположению (ii) имеем | Р | 2 < r . Из уравнений ( 12)–( 14) имеем Это противоречие и условие (а) в предложении 4.1 выполнено.

Далее мы покажем, что выполняется гипотеза (b) предложения 4.1. Необходимо рассмотреть следующие два случая: случаи λ 1 (0)<1<λ 2 (0) и 1<λ 1 (0)≤λ 2 (0). Заметим, что если λ 1 (0)<1<λ 2 (0), то 1− T 0 + D 0 <0, а если 1<λ 1 (0)≤λ 2 (0), то 1− T 0 + D 0 9032 1 > 0. По непрерывности можно взять достаточно малое τ∈(0, σ) такое, что если | Р | 2 ≤τ, затем 1− T P + D P <0 и λ 2 ( 90 005 P )>1 выполняется в случае λ 1 (0 )<1<λ 2 (0), и 1− T P + D P >0 и λ 2 ( P )>1 выполняется в случае 1<λ 1 (0)≤λ 2 (0).

Сначала рассмотрим случаи λ 1 (0)<1<λ 2 (0) и . В этом случае пусть . Чтобы показать, что выполняется гипотеза (b) предложения 4.1, предположим, что существует λ>0 и ( P ,  a )∈ C σ такое, что |( P ,  a )| 4 =τ и ( P ,  a )− f ( P ,  a )=λ x 1 . Затем у нас есть где I — единичная матрица. Отсюда имеем Поскольку 1− T P + D P < 0, из уравнений (15) и (16) следует, что и . Но тогда, а это противоречит 1− T P + D P <0.

Во-вторых, рассмотрим случаи λ 1 (0)<1<λ 2 (0) и . Предположим, что существует λ>0 и ( P ,  a )∈ C σ такое, что |( P ,  a )| 4 =τ и ( P ,  a )− f ( P ,  a )=λ x 1 . Тогда имеем Из этих уравнений имеем С 1− T P + D P <0, уравнение (17) подразумевает, что . Тогда a 2 >0 и по уравнению (18) имеем , что является противоречием.

В-третьих, рассмотрим случай 1<λ 1 (0)≤λ 2 (0). Предположим, что существует λ>0 и ( P ,  a )∈ C σ такое, что |( P ,  a )| 4 =τ и ( P ,  a )− f ( P ,  a )=λ х 1 . Тогда у нас есть Вспомните, что . Если , то матрица положительно определена, что противоречит уравнению (19). Если , то имеем . В самом деле , а значит, . Тогда уравнение (19) переписывается в виде где Поскольку положительно определено и , уравнение (20) дает противоречие.

Следовательно, условие (b) в предложении 4.1 выполняется в любом случае. Таким образом, по предложению 4.1 существует ( P * ,  a * )∈ C σ такое, что f ( P * ,  a * )=( P * ,  a * ) и ( P * ,  a * )≠(0, 0). Это завершает доказательство.

Замечание 4.3

В случае взаимодействия видов, как в примере 1.1, мы принимаем if i j . Тогда легко видеть, что (или λ 2 ( P ) соответственно) и (или λ 1 ( P ) соответственно). Пусть и . Тогда предположения (i) и (ii) в теореме 4.2 читаются как , , и при достаточно больших | P |> r . Таким образом, теорема 4.2 совместима с Уэббом [Citation11, теорема 4.1], и, грубо говоря, теорема 4.2 утверждает, что если один из двух видов удовлетворяет условию, что чистый коэффициент воспроизводства больше 1, когда общая численность популяции равна нулю, и меньше 1, когда общая численность населения достаточно велика, то существует ненулевое стационарное решение.

временные ряды. Нестационарные решения для стационарных уравнений ARMA

спросил

Изменено
2 года, 4 месяца назад

Просмотрено
970 раз

$\begingroup$

Под «стационарным» я подразумеваю «слабо стационарный».

9t$, которое оказывается нестационарным решением «стационарного» уравнения (очевидно, что $\mathbb{E}[Y_t]$ не свободен от $t$, так как $X_t$, очевидно, имеет нулевое среднее).

Можно ли каким-то образом нарушить свойство слабой стационарности при более общем стационарном AR($p$) процессе? Или, вообще говоря, верно ли, что любое стационарное уравнение AR (или даже ARMA) с дискретным временем имеет нестационарное решение?

  • временной ряд
  • арима
  • стационарность
  • авторегрессия

$\endgroup$

5

$\begingroup$

Терминология, использованная в вопросе, не совсем корректна. Вы смешиваете модель (или уравнения) и решение модели.

Не имеет смысла говорить об уравнении (в данном случае о системе стохастических разностных уравнений), стационарном или нестационарном. {-p})$ не имеет корней на единичной окружности и все корни действительны. 9t$ является нестационарным решением уравнения $\text{AR}(1)$. Таким образом, вопреки вашей терминологии, это не «стационарное уравнение», и оно не «повреждается» наличием нестационарного решения. Нестационарное решение — это просто нестационарная модель, представляющая собой форму процесса $\text{AR}(1)$, согласующегося с рекурсивными уравнениями. Существуют также нестационарные решения уравнений $\text{AR}(p)$.

$\endgroup$

$\begingroup$

9т]=_{т\к\infty}0$$
это несмотря на то, что правая часть зависит от конечных $t$.

Таким образом, ответ на ваш вопрос заключается в том, что ваш процесс $Y_t$ , а не нестационарный. Следовательно, это не служит контрпримером.

Дополнительные мысли .
Вы сформулировали свой вопрос в терминах решений случайных процессов.

ООО "ПАРИТЕТ" © 2021. Все права защищены.